You are on page 1of 7

First selection test, May 1st , 2008

Problem 1. Let p be a prime number, p 6= 3, and let a, b be integer numbers


so that p | a + b and p2 | a3 + b3 . Show that p2 | a + b or p3 | a3 + b3 .
Problem 2. Prove that for any positive integer n there exists a multiple of
n with the sum of its digits equal to n.
Mihai Bălună
Problem 3. Let ABC be an acute-angled triangle. Consider the equilateral
triangle A0 U V , with A0 ∈ (BC), U ∈ (AC), V ∈ (AB) such that U V k BC.
Similarly, define points B 0 ∈ (AC) and C 0 ∈ (AB). Show that the lines AA0 ,
BB 0 and CC 0 are concurrent.
Vasile Pop
Problem 4. Let ABC be a triangle and D the midpoint of BC. On the

L
sides AB and AC there are points M , N respectively, other than the midpoints

.M
of these segments, so that AM 2 + AN 2 = BM 2 + CN 2 and ∠M DN = ∠BAC.
Prove that A = 90◦ .
Francisc Bozgan

D
Problem 5. Let n ∈ N, n ≥ 2. Consider the integers a1 , a2 , . . . an with
A
0 < ak ≤ k, for all k = 1, 2, . . . , n. Given that a1 + a2 + . . . + an is an even
PI
number, prove that one can choose the signs ‘+’ and ‘−’ so that

a1 ± a2 ± . . . ± an = 0.
M
LY

Second selection test, June 5th , 2008


O

Problem 6. Consider an acute-angled triangle ABC, the height AD and


.M

the point E where the diameter from A of the circumcircle intersects the line
BC. Let M, N be the mirror images of D across the lines AC and AB. Show
that ∠EM C = ∠BN E.
W

Dinu Şerbănescu
W

Problem 7. A sequence of integers a1 , a2 , . . . , an is given so that ak is the


number of all multiples of k in this sequence, for any k = 1, 2, . . . , n. Find all
W

possible values for n.


Cristian Mangra
Problem 8. Let n ∈ N∗ and let a1 , a2 , . . . , an be positive real numbers so
that
1 1 1
a1 + a2 + . . . + an = 2 + 2 + . . . + 2 .
a1 a2 an
Prove that for any m = 1, 2 . . . , n, there exist m numbers among the given ones
with the sum not less than m.
Andrei Ciupan & Flavian Georgescu
Problem 9. Let a, b be real numbers with the property that the integer
part of an + b is an even number, for all n ∈ N. Show that a is an even integer.
Dinu Şerbănescu

1
Third selection test, June 6th , 2008

Problem 10. Ten numbers are chosen at random from the set 1,2,3,...,37.
Show that one can select four distinct numbers from the chosen ones so that
the sum of two of them is equal to the sum of the other two.
Vasile Pop
Problem 11. Let a, b, c be positive real numbers with ab + bc + ca = 3.
Prove that
1 1 1 1
+ + ≤ .
1 + a2 (b + c) 1 + b2 (c + a) 1 + c2 (a + b) abc

Vlad Matei
Problem 12. Find all primes p, q satisfying the equation 2pq − q p = 7.

L
Francisc Bozgan

.M
Problem 13. Let d be a line and let M, N be two points on d. Circles
α, β, γ, δ centered at A, B, C, D are tangent to d in such a manner that circles
α, β are externally tangent at M , while circles γ, δ are externally tangent at

D
N . Moreover, points A and C lies on the same side of line d. Prove that if
A
there exists a circle tangent to all circles α, β, γ, δ, containing all of them in the
interior, then lines AC, BD and d are concurrent or parallel.
PI
Flavian Georgescu
M

Fourth selection test, June 9th , 2008


LY

Problem 14. Let ABCD be a quadrilateral with no two opposite sides


O

parallel. The parallel from A to BD meets the line CD at point F and the
parallel from D at AC meet the line AB at point E. Consider the midpoints
.M

M, N, P, Q of the segments AC, BD, AF, DE respectively. Show that lines


M N, P Q and AD are concurrent.
Dinu Şerbănescu
W

Problem 15. Let m, n ∈ N∗ and A = {1, 2, . . . , n}, B = {1, 2, . . . , m}.


W

A subset S of the set product A × B has the property that for any pairs
(a, b), (x, y) ∈ S, then (a − x)(b − y) ≤ 0. Show that S has at most m + n − 1
W

elements.
Dinu Şerbănescu
Problem 16. Find all pairs of integers (m, n), n, m > 1 so that mn − 1
divides n3 − 1.
Francisc Bozgan
Problem 17. Determine the maximum value of the real number k such
that  
1 1 1
(a + b + c) + + − k ≥ k,
a+b b+c a+c
for all real numbers a, b, c ≥ 0 with a + b + c = ab + bc + ca.
Andrei Ciupan

2
SOLUTIONS

Problem 1.
Suppose that p2 - a + b. It suffices to prove that p3 | a3 + b3 . Indeed, if
p | (a + b)3 − 3ab(a + b), we have p | 3ab. As p 6= 3 is prime, it follows that
2

p | a or p | b. Since p | a + b, we get p | a and p | b. As a consequence, p3 | a3


and p3 | b3 , implying p3 | a3 + b3 .
Problem 2. Let n ≥ 1 and let 10k , k ∈ N. Consider all the remainders of the
numbers 10k at the division by n. Since there are only finitely many residues,
there exists a = 0, 1, . . . , n − 1 so that 10m ≡ a (mod n) for infinitely many
values of m ∈ N. Among them select only n, namely 10m1 , 10m2 , . . . , 10mn ,
with m1 > m2 > . . . > mn . The number A = 10m1 + 10m2 + . . . + 10mn has n
digits equal to 1 and the rest equal to 0, has the sum of all digits is n. Moreover,
A ≡ na ≡ 0 (mod n), so n | A, which concludes the proof.

L
.M
Problem 3. Consider the equilateral triangle BCA1 , constructed in the
exterior of the triangle ABC. Then points A, A0 , A1 are collinear, through the
homothety of center A which map points U, V in B, C, respectively.

D
Since AA1 , BB1 , CC1 concur in the Fermat-Torricelli point of the triangle
ABC, the claim is proved.
A
PI
Problem 4. Let E and F be the midpoint of the sides AC and AB and let
P be the mirror image of D across E.
The relation AM 2 + AN 2 = BM 2 + CN 2 gives ( 2c + F M )2 + ( 2b − N E)2 =
M

FM NE
( 2c − F M )2 + ( 2b + N E)2 , hence c · F M = b · N E. Then = , so
LY

AC AB
FM NE
= . Since ∠M F D = ∠N EP , we get ∆M F D ∼ ∆N EP , which
FD EP
O

implies ∠M DF = ∠N P E. On the other hand ∠M DN = ∠BAC = ∠F DE, so


∠M DF = ∠N DE.
.M

Now the triangle N P D is isosceles and N E is a median in this triangle, so


N E⊥DP , in other words A = 90◦ .
Problem 5. Consider An−1 = an − an−1 . Since an ≤ n and an−1 ≥ 1, we
W

have An−1 ≤ n − 1.
W

If An−1 = 0, that is an−1 = an , then a1 + a2 + . . . + an−2 is even and the


claim reduces to the case of n − 2 numbers.
W

If An−1 > 0, then a1 + a2 + . . . + an−2 + An−1 is even and the claim reduces
to the case of n − 1 numbers.
Problem 6. Observe that AD = AN = AM and ∠AN D = ∠AM C =
90◦ , due to the reflections across AB and AC. It is known that AD and AE
are isogonal cevians, that is ∠BAD = ∠EAC. Then ∠N AE = ∠N AB +
∠BAE = ∠BAD + ∠BAE = ∠EAC + ∠DAC = ∠EAC + ∠CAM = ∠EAM
and consequently ∆N AE ≡ ∆EAM . It follows that ∠EN A = ∠EM A, so
∠BN E = 90◦ − ∠EN A = 90◦ − ∠EM A = ∠EM C, as desired.
Problem 7. Notice that a1 = n, as 1 divides any aj , and aj ≤ n, for any
j = 1, 2, . . . , n.
Consider an array with rows corresponding to 1,2,...,n and columns corre-
sponding to the numbers a1 , a2 , . . . , an . Define the entries as follows: on the
position (i, aj ) put 1 if i divides aj and 0 otherwise. Now observe that the sum
of the numbers placed of the ith row is equal to ai , as we see the entries 1 for

3
each multiple ofPi in the sequence a1 , a2 , . . . , an . Hence the sum of all the entries
in the array is aj .
On the other hand, on the athj column we get and entry 1 as long as we count
the divisors of aj , so the sum of the numbers on the j th columns is the number
of divisors of aj . This implies that the sum of all the entries in the array is the
sum of all divisors of the numbers aj .
Using this double-counting, together with the obvious fact that the number
of divisor of a is less than a - unless a is 1 or 2, show that n = 1 or n = 2.
Alternative solution. Recall that a1 = n and ai ≤ n, for all i = 1, 2, . . . , n.
Assume that n > 3. As an−1 ≥ 1, the exists a multiple of n − 1, where n − 1 > 1,
in the given sequence; let ak , k > 1 be such a multiple. The condition ai ≤ n
shows that ak = n − 1, in other words there are n − 1 multiples of k in the
sequence. As n and n − 1 are coprime, k does not divide a1 = n, so k divides
a2 , . . . , an . But k ≥ 2 and k | an , therefore an > 1. Thus n must occur at
least twice in the sequence, so, beside a1 we have aj = n, j > 1. Hence k | n, a

L
contradiction. As before, n = 1 or n = 2 are the only possible values.

.M
Problem 8. It is clear that we need to prove that a1 + a2 + . . . + an ≥ n. Let
us notice that this is enough: let m = 1, 2, . . . , n and assume that any selection

D
of m numbers from the given ones has the sum less than 1. Then add the n
inequalities
a1 + a2 + . . . + am < m, A
PI
a2 + a3 + . . . + am+1 < m,
M

..
.
LY

an + a1 + . . . + am−1 < m,
to get n(a1 + a2 + . . . + an ) < nm, which is a contradiction.
O

Back to the top, let g be the geometrical mean of the numbers a1 , a2 , . . . , an


and suppose that a1 + a2 + . . . + an < n.
.M

a1 + a2 + . . . + an
By AM-GM inequality, we have g ≤ < 1, while 1 >
n
1 1 1
a1 + a2 + . . . + an a2
+ a2 + . . . + a2 1
n
W

= 1 2
≥ 2 , which gives g > 1, a contradic-
n n g
tion.
W

Problem 9. Let [an + b] = 2xn , for all integers n > 0. Then


W

2xn ≤ an + b < 2xn + 1, (1)


2xn+1 ≤ a(n + 1) + b < 2xn+1 + 1. (2)

Subtracting (1) from (2) we get 2(xn+1 − xn ) − 1 < a < 2(xn+1 − xn ) + 1,


for all n > 0. As 2(xn+1 − xn ) − 1 is an odd integer, it follows that all numbers
2(xn+1 − xn ) − 1 must be equal, as otherwise a belongs to two open intervals of
lengths 2 having the left margins at a difference of at least 2, which is impossible.
Hence 2(xn+1 − xn ) − 1 = 2s − 1, s ∈ Z, so xn+1 − xn = s and then xn = ns + p,
p ∈ Z, ∀n > 0.
Plugging in (1) we get 2p−b ≤ (a−2s)n < 2p−b+1, ∀n > 0, so a = 2s, since
otherwise the set of the positive integers will have an upper margin. Observe
that s is an integer, so a is an even integer, as needed.
Problem 10. Consider all positive differences a − b among all 10 numbers.
2
Since there are C10 = 45 positive differences and all belong to the set 1,2,3,...,36,

4
at least two of them are equal. Let them be a − b and c − d, with a > c. If
a, b, c, d are all distinct, we are done; if not, then b = c, so b = c is one of the 8
numbers which are neither the lowest nor the greatest number from the initial
ones.
Now observe that we have 45 positive differences and 36 possible values for
them, so either 3 positive differences are equal or there are 9 pairs of equal
positive differences.
The first case gives a − b = c − d = e − f , with a > c > e. Since we cannot
have b = c, b = e and d = e, we are done.
The second case gives at least one pair of positive differences in which case
b = c is excluded, as only 8 candidates for b = c exist, so we are done.
ab + bc + ca
Problem 11. Using the AM-GM inequality we derive ≥
p 3
3
(abc)2 . As ab + bc + ca = 3, then abc ≤ 1. Now

L
X 1 X 1 X 1
= =

.M
1+ a2 (b
+ c) 1 + a(ab + ac) 1 + a(3 − bc)
X 1 X 1 ab + bc + ca 1
= ≤ = = , as claimed.

D
3a + (1 − abc) 3a 3abc abc

A
Problem 12. It is easy to observe that p is odd and p 6= q, in other words
PI
p ≥ 3 and (p, q) = 1.
If q = 2, then 2p+1 = 7 + p2 . The only solution is p = 3, as 2n+1 > 7 + n2 , for
M

all n ≥ 4. For q ≥ 3, by Little Fermat’s Theorem we get p | 2q − 7 and q | p + 7.


Set p + 7 = kq, k ∈ N∗ .
LY

If 2q − 7 ≤ 0, we have q = 3 and p | −1, false.


If 2q − 7 > 0, then 2q − 7 ≥ p, so 2q ≥ p + 7 ≥ kq, therefore k = 1 or k = 2.
For k = 1 we obtain p + 7 = q, so p | 2p + 7. This implies p = 7 and then q = 14,
O

false. Hence k = 2 and p + 7 = 2q. Suppose p > q; as p, q ≥ 3 we get q p ≥ q p


and then 7 = 2q p − pq ≥ q p ≥ 33 = 27, a contradiction. Thus q > p and then
.M

p + 7 = 2q > 2p, which yields p = 3 or p = 5. For p = 3 we have q = 5, while


p = 5 gives q | 12, with no solution.
W

To conclude, the solutions are (p, q) = (3, 2), (3, 5).


Problem 13. Let a, b, c, d be the radii of the circles α, β, γ, δ. It suffices to
W

prove that ab = dc , in other words the ratio ab is constant while point M varies
on line d.
W

Let R and S be the midpoints of the arcs determined by d on the fifth circle
K, the one tangent simultaneously to α, β, γ, δ, and let N be on the same side of
d as A. Denote by A1 and B1 the tangency point of α and β to K, respectively.
Observe that points A1 , M, R are collinear - via the homothety which maps
circle α onto circle K - and similarly points B1 , M, S are collinear. Since RS is
a diameter of K, angles ∠RA1 S and ∠SB1 R are right. If lines B1 R and A1 S
meet as point V , then M is the orthocenter of the triangle V RS. Notice that
d⊥RS, hence V ∈ d; denote by O the intersection point of d and RS.
Lines A1 S and B1 R intersect the circles α and β at points U and Z re-
spectively. Since ∠RA1 S = ∠SB1 R = 90◦ , the segments U M and ZM are
diameters in circles α, β, so ab = U M RO
ZM = SO . The latter ratio is constant, as
claimed.
Problem 14. Let O be the midpoint of AD, R be the intersection point of
lines AC and BD and S be the intersection point of lines AF and DE. Since
N and Q are the midpoints of the sides DB and DE of the triangle DBE, we

5
have O ∈ N Q and similarly O ∈ M P . Moreover, as DRAS is a parallelogram,
the diagonal RS passes through the midpoint O of the other diagonal, AD.
Now apply Desargues Theorem for triangles N RM and SP Q, given that O lies
simultaneously on lines N Q, M P, RS and we are done.
Problem 15. Consider a set S which satisfies all requirements. For each
i ∈ A = {1, 2, . . . , n}, define Bi ⊂ B the set of all elements j ∈ B for which
the pair (i, j) belongs to the set S - notice that some subsets Bi can be empty.
Counting all pairs in S over all second element in each pair, we have |S| =
|B1 | + |B2 | + . . . + |Bn |.
The main idea is to observe the chain of ‘inequalities’

B1 ≤ B2 ≤ · · · ≤ Bn ,

where by X ≤ Y we mean that x ≤ y, for any x ∈ X and y ∈ Y , X, Y being


sets of integers. (This definition allows the empty set to occupy any position in

L
this chain).

.M
Since B1 ∩ B2 ∩ . . . ∩ Bn = B = {1, 2, . . . , m} and any two consecutive
subsets Bi share in common at most one element, we get - by sieve thorem -
that |S| ≤ m + n − 1, as claimed.

D
Problem 16. The solutions are (k, k 2 ) and (k 2 , k), with k > 1.
A
We have mn − 1 | (n3 − 1)m − n2 (mn − 1) = n2 − m. On the other hand,
PI
mn − 1 | m(n2 − m) − (mn − 1)n = n − m2 .
If n > m2 , then mn − 1 ≤ n − m2 ≤ n − 1, so mn ≤ n, false.
M

If n = m2 , then obviously m3 − 1 | m6 − 1, so all pairs (m, m2 ), m > 1 are


solutions. √
LY

If n < m2 , from mn − 1 ≤ n3 − 1 we derive that n < m ≤ n2 . Then


mn − 1 ≤ m2 − n < m2 − 1, so n < m. If n2 − m > 0, we obtain mn − 1 ≤
O

n2 − m < n2 − 1, so m < n, a contradiction. Hence n = m2 , which holds, since


m3 − 1 | m3 − 1, so all pairs (n2 , n), n > 1 are also solutions.
.M

Problem 17. Observe that the numbers a = b = 2, c = 0 fulfill the


condition
 ab + bc + ca =a + b + c. Plugging into the given inequality, we derive
1 1 1
W

that 4 + + − k ≥ k, hence k ≤ 1.
4 2 2
We claim that the inequality holds for k = 1, proving that the maximum
W

value of k is 1. For this, rewrite the inequality as


W

 
1 1 1
(ab + bc + ca) + + −1 ≥1⇔
a+b b+c a+c
X ab + bc + ca
≥ ab + bc + ca + 1 ⇔
a+b
 
X ab X ab
+ c ≥ ab + bc + ca + 1 ⇔ ≥ 1.
a+b a+b

ab ab
Notice that ≥ , since a, b, c ≥ 0. Summing over a cyclic
a+b a+b+c
permutation of a, b, c we get
X ab X ab ab + bc + ca
≥ = = 1,
a+b a+b+c a+b+c
as nedeed.

6
Alternative solution. The inequality is equivalent to the following:
 
a+b+c 1 1 1
S= + + ≥ k.
a+b+c+1 a+b b+c a+c

Using the given condition, we get

1 1 1
+ +
a+b b+c a+c
a2 + b2 + c2 + 3(ab + bc + ca)
=
(a + b)(b + c)(c + a)
2 2 2
a + b + c + 2(ab + bc + ca) + a + b + c
=
(a + b + c)(ab + bc + ca) − abc
(a + b + c)(a + b + c + 1)

L
= ,
(a + b + c)2 − abc

.M
hence
(a + b + c)2
S= .

D
(a + b + c)2 − abc

A
Now it is clear that S ≥ 1, and the equality occurs for abc = 0. Therefore
k = 1 is the maximum value.
PI
M
LY
O
.M
W
W
W

You might also like